Witt ring of field

The name of the pictureThe name of the pictureThe name of the pictureClash Royale CLAN TAG#URR8PPP











up vote
1
down vote

favorite
1












Why $W(C)$ is isometric to $mathbbZ_2$ and $W(R)=mathbbZ$ how to get this explicitly? I know we can count $mathbbC$ as even dimensional space over $mathbbR$ so hyperbolic space hence it is $mathbbZ_2$ but I can't get why it is true explicitly? Also for algebraically closed field it is isomorphic to $mathbbZ_2$ please explain this also.







share|cite|improve this question


























    up vote
    1
    down vote

    favorite
    1












    Why $W(C)$ is isometric to $mathbbZ_2$ and $W(R)=mathbbZ$ how to get this explicitly? I know we can count $mathbbC$ as even dimensional space over $mathbbR$ so hyperbolic space hence it is $mathbbZ_2$ but I can't get why it is true explicitly? Also for algebraically closed field it is isomorphic to $mathbbZ_2$ please explain this also.







    share|cite|improve this question
























      up vote
      1
      down vote

      favorite
      1









      up vote
      1
      down vote

      favorite
      1






      1





      Why $W(C)$ is isometric to $mathbbZ_2$ and $W(R)=mathbbZ$ how to get this explicitly? I know we can count $mathbbC$ as even dimensional space over $mathbbR$ so hyperbolic space hence it is $mathbbZ_2$ but I can't get why it is true explicitly? Also for algebraically closed field it is isomorphic to $mathbbZ_2$ please explain this also.







      share|cite|improve this question














      Why $W(C)$ is isometric to $mathbbZ_2$ and $W(R)=mathbbZ$ how to get this explicitly? I know we can count $mathbbC$ as even dimensional space over $mathbbR$ so hyperbolic space hence it is $mathbbZ_2$ but I can't get why it is true explicitly? Also for algebraically closed field it is isomorphic to $mathbbZ_2$ please explain this also.









      share|cite|improve this question













      share|cite|improve this question




      share|cite|improve this question








      edited Aug 16 at 8:39









      Bernard

      111k635103




      111k635103










      asked Aug 16 at 8:26









      Ninja hatori

      149113




      149113




















          1 Answer
          1






          active

          oldest

          votes

















          up vote
          0
          down vote













          Calculations of the Witt ring are way too demanding for simple Math Stackexchange answer. I'll give you hints and resources.



          For a field $k$ of $textchar(k)neq 2$ the following are equivalent:



          • $k$ does not admit quadratic field extension

          • $W(k)simeqmathbbZ_2$

          You can find the proof in the following paper (Proposition 11):



          https://pdfs.semanticscholar.org/a1c5/a6eb6f12a8dfd0f1f01cf57d6b4fa4233beb.pdf



          Now since algebraically closed fields have no finite extensions then in particular they don't have quadratic field extensions so their Witt ring is $mathbbZ_2$.



          For reals $mathbbR$ read the same paper, Proposition 26. I actually encourage you to read the whole paper.






          share|cite|improve this answer






















          • math.stackexchange.com/questions/2888950/…
            – Ninja hatori
            Aug 22 at 11:19










          • paper doesn't included proof for most of theorems.
            – Ninja hatori
            Aug 22 at 11:19










          Your Answer




          StackExchange.ifUsing("editor", function ()
          return StackExchange.using("mathjaxEditing", function ()
          StackExchange.MarkdownEditor.creationCallbacks.add(function (editor, postfix)
          StackExchange.mathjaxEditing.prepareWmdForMathJax(editor, postfix, [["$", "$"], ["\\(","\\)"]]);
          );
          );
          , "mathjax-editing");

          StackExchange.ready(function()
          var channelOptions =
          tags: "".split(" "),
          id: "69"
          ;
          initTagRenderer("".split(" "), "".split(" "), channelOptions);

          StackExchange.using("externalEditor", function()
          // Have to fire editor after snippets, if snippets enabled
          if (StackExchange.settings.snippets.snippetsEnabled)
          StackExchange.using("snippets", function()
          createEditor();
          );

          else
          createEditor();

          );

          function createEditor()
          StackExchange.prepareEditor(
          heartbeatType: 'answer',
          convertImagesToLinks: true,
          noModals: false,
          showLowRepImageUploadWarning: true,
          reputationToPostImages: 10,
          bindNavPrevention: true,
          postfix: "",
          noCode: true, onDemand: true,
          discardSelector: ".discard-answer"
          ,immediatelyShowMarkdownHelp:true
          );



          );








           

          draft saved


          draft discarded


















          StackExchange.ready(
          function ()
          StackExchange.openid.initPostLogin('.new-post-login', 'https%3a%2f%2fmath.stackexchange.com%2fquestions%2f2884545%2fwitt-ring-of-field%23new-answer', 'question_page');

          );

          Post as a guest






























          1 Answer
          1






          active

          oldest

          votes








          1 Answer
          1






          active

          oldest

          votes









          active

          oldest

          votes






          active

          oldest

          votes








          up vote
          0
          down vote













          Calculations of the Witt ring are way too demanding for simple Math Stackexchange answer. I'll give you hints and resources.



          For a field $k$ of $textchar(k)neq 2$ the following are equivalent:



          • $k$ does not admit quadratic field extension

          • $W(k)simeqmathbbZ_2$

          You can find the proof in the following paper (Proposition 11):



          https://pdfs.semanticscholar.org/a1c5/a6eb6f12a8dfd0f1f01cf57d6b4fa4233beb.pdf



          Now since algebraically closed fields have no finite extensions then in particular they don't have quadratic field extensions so their Witt ring is $mathbbZ_2$.



          For reals $mathbbR$ read the same paper, Proposition 26. I actually encourage you to read the whole paper.






          share|cite|improve this answer






















          • math.stackexchange.com/questions/2888950/…
            – Ninja hatori
            Aug 22 at 11:19










          • paper doesn't included proof for most of theorems.
            – Ninja hatori
            Aug 22 at 11:19














          up vote
          0
          down vote













          Calculations of the Witt ring are way too demanding for simple Math Stackexchange answer. I'll give you hints and resources.



          For a field $k$ of $textchar(k)neq 2$ the following are equivalent:



          • $k$ does not admit quadratic field extension

          • $W(k)simeqmathbbZ_2$

          You can find the proof in the following paper (Proposition 11):



          https://pdfs.semanticscholar.org/a1c5/a6eb6f12a8dfd0f1f01cf57d6b4fa4233beb.pdf



          Now since algebraically closed fields have no finite extensions then in particular they don't have quadratic field extensions so their Witt ring is $mathbbZ_2$.



          For reals $mathbbR$ read the same paper, Proposition 26. I actually encourage you to read the whole paper.






          share|cite|improve this answer






















          • math.stackexchange.com/questions/2888950/…
            – Ninja hatori
            Aug 22 at 11:19










          • paper doesn't included proof for most of theorems.
            – Ninja hatori
            Aug 22 at 11:19












          up vote
          0
          down vote










          up vote
          0
          down vote









          Calculations of the Witt ring are way too demanding for simple Math Stackexchange answer. I'll give you hints and resources.



          For a field $k$ of $textchar(k)neq 2$ the following are equivalent:



          • $k$ does not admit quadratic field extension

          • $W(k)simeqmathbbZ_2$

          You can find the proof in the following paper (Proposition 11):



          https://pdfs.semanticscholar.org/a1c5/a6eb6f12a8dfd0f1f01cf57d6b4fa4233beb.pdf



          Now since algebraically closed fields have no finite extensions then in particular they don't have quadratic field extensions so their Witt ring is $mathbbZ_2$.



          For reals $mathbbR$ read the same paper, Proposition 26. I actually encourage you to read the whole paper.






          share|cite|improve this answer














          Calculations of the Witt ring are way too demanding for simple Math Stackexchange answer. I'll give you hints and resources.



          For a field $k$ of $textchar(k)neq 2$ the following are equivalent:



          • $k$ does not admit quadratic field extension

          • $W(k)simeqmathbbZ_2$

          You can find the proof in the following paper (Proposition 11):



          https://pdfs.semanticscholar.org/a1c5/a6eb6f12a8dfd0f1f01cf57d6b4fa4233beb.pdf



          Now since algebraically closed fields have no finite extensions then in particular they don't have quadratic field extensions so their Witt ring is $mathbbZ_2$.



          For reals $mathbbR$ read the same paper, Proposition 26. I actually encourage you to read the whole paper.







          share|cite|improve this answer














          share|cite|improve this answer



          share|cite|improve this answer








          edited Aug 17 at 9:11

























          answered Aug 17 at 9:05









          freakish

          8,6971524




          8,6971524











          • math.stackexchange.com/questions/2888950/…
            – Ninja hatori
            Aug 22 at 11:19










          • paper doesn't included proof for most of theorems.
            – Ninja hatori
            Aug 22 at 11:19
















          • math.stackexchange.com/questions/2888950/…
            – Ninja hatori
            Aug 22 at 11:19










          • paper doesn't included proof for most of theorems.
            – Ninja hatori
            Aug 22 at 11:19















          math.stackexchange.com/questions/2888950/…
          – Ninja hatori
          Aug 22 at 11:19




          math.stackexchange.com/questions/2888950/…
          – Ninja hatori
          Aug 22 at 11:19












          paper doesn't included proof for most of theorems.
          – Ninja hatori
          Aug 22 at 11:19




          paper doesn't included proof for most of theorems.
          – Ninja hatori
          Aug 22 at 11:19












           

          draft saved


          draft discarded


























           


          draft saved


          draft discarded














          StackExchange.ready(
          function ()
          StackExchange.openid.initPostLogin('.new-post-login', 'https%3a%2f%2fmath.stackexchange.com%2fquestions%2f2884545%2fwitt-ring-of-field%23new-answer', 'question_page');

          );

          Post as a guest













































































          這個網誌中的熱門文章

          Is there any way to eliminate the singular point to solve this integral by hand or by approximations?

          Why am i infinitely getting the same tweet with the Twitter Search API?

          Carbon dioxide